Strengthen with necessary premise
Could you please explain this answer in detail? Also, do you have any suggestions on how to answ...
RKHanda13 on November 26, 2013
  • October 2008 LSAT
  • SEC3
  • Q19
1
Reply
Map
I did not map this out effectively. Can I have some help?
Nathan on November 25, 2013
  • June 2000 LSAT
  • SEC2
  • Q18
1
Reply
Principle
Why is the answer b and not e?
RKHanda13 on November 21, 2013
  • October 2008 LSAT
  • SEC1
  • Q21
1
Reply
Q1: help
Can someone please explain this? :)
Deniann on November 21, 2013
  • September 2009 LSAT
  • SEC2
  • Q1
1
Reply
Flawed parallel reasoning
Could you please explain the reasoning behind this answer?
RKHanda13 on November 20, 2013
  • October 2008 LSAT
  • SEC1
  • Q1
1
Reply
Question
Can someone please explain this?
Deniann on November 16, 2013
  • September 2009 LSAT
  • SEC1
  • Q19
1
Reply
Diagnostic2, question 13
Can someone please explain this? :)
Deniann on November 15, 2013
  • September 2009 LSAT
  • SEC1
  • Q13
1
Reply
Lifestyle and heart disease
First thing I noticed: Correlation [~]btw high cholesterol and high risk of death due to heart a...
Lily on November 11, 2013
  • October 1999 LSAT
  • SEC1
  • Q6
1
Reply
Eighteenth-century moralist
Please provide a detailed explanation why the answer is E.
RKHanda13 on November 11, 2013
  • October 2008 LSAT
  • SEC1
  • Q14
1
Reply
Confused
Is there an easier way/ any tricks that I could use to break down what B is saying? The language ...
Deniann on November 8, 2013
  • December 2004 LSAT
  • SEC2
  • Q23
1
Reply
Question 20
Can someone please explain this?
Deniann on November 6, 2013
  • December 2004 LSAT
  • SEC2
  • Q19
1
Reply
Help
Why is answer choice E the correct answer?
Alex07 on November 4, 2013
  • October 2004 LSAT
  • SEC4
  • Q21
1
Reply
#113
Why can't the order be g,j,m,g,j,t,m? Wouldn't that fit the rules and have t in week 6?
Mreid on October 6, 2013
  • June 2007 LSAT
  • SEC1
  • Q12
1
Reply
Explanation
Can someone explain to me how A is the correct answer? Thank you!
Joesolo on October 4, 2013
  • December 2010 LSAT
  • SEC2
  • Q15
1
Reply
Citations to students
I usually never get this type wrong, but I cannot wrap my head around this one!
Lily on October 1, 2013
  • October 2011 LSAT
  • SEC1
  • Q24
1
Reply
This game
I can't figure out how to set it up.
Aclsat on September 24, 2013
  • December 2012 LSAT
  • SEC4
  • Q11
2
Replies
Why is the answer A?
Confused as to why A is the correct answer.
Dlinu110 on September 17, 2013
  • June 1994 LSAT
  • SEC4
  • Q19
1
Reply
Soft drink survey error
On the print out version, this game 7-12 and the next game about benches 13-19 are misplaced :)
Lily on September 10, 2013
  • October 1992 LSAT
  • SEC4
  • Q7
1
Reply
Cold medication rankings
So I got these right, but took a bit longer than expected ... Wondering if I could have used the ...
Lily on September 9, 2013
  • June 2003 LSAT
  • SEC2
  • Q6
3
Replies
Unicorns?
I chose answer 'E'. Why was this wrong and 'A' right.
Nathan on August 31, 2013
  • September 2007 LSAT
  • SEC1
  • Q16
2
Replies